Đề thi và đáp án kì thi chọn đội tuyển thi Quốc gia trường Phổ thông Năng khiếu năm học 2018 – 2019

ĐỀ THI

Ngày thi thứ nhất

Bài 1. Cho số nguyên $a>1$. Tìm giá trị lớn nhất của số thực $d$ sao cho tồn tại một cấp số cộng có công sai $d$, số hạng đầu tiên là $a$ và có đúng hai trong các số $a^2, a^3, a^4, a^5$ là những số hạng của cấp số cộng đó.

Bài 2. Cho $n$ số thực $x_1, x_2, \ldots, x_n$. Với mỗi $i \in{1,2, \ldots, n}$, gọi $a_i$ là số các chỉ số $j$ mà $\left|x_i-x_j\right| \leq 1$ và $b_i$ là số các chỉ số $j$ mà $\left|x_i-x_j\right| \leq 2$ ( $i$ và $j$ có thể bằng).

(a) Chứng minh rằng tồn tại $i$ để $b_i \leq 3 a_i$.

(b) Gọi $A$ là số cặp $(i, j)$ có thứ tự mà $\left|x_i-x_j\right| \leq 1$ và $B$ là số cặp $(i, j)$ có thứ tự mà $\left|x_i-x_j\right| \leq 2$ ( $i$ và $j$ có thể bằng nhau). Chứng minh rằng $B \leq 3 A$.

Bài 3. Cho $p$ là số tự nhiên. Xét phương trình nghiệm nguyên $x^3+x+p=y^2$.

(a) Tìm số nguyên tố $p$ nhỏ nhất dạng $4 k+1$ sao cho phương trình có nghiệm.

(b) Chứng minh rằng nếu $p$ là số chính phương thì phương trình trên có nghiệm nguyên dương.

Bài 4. Cho tam giác $A B C$ nhọn nội tiếp đường tròn $(O)$ với $B, C$ cố định và $A$ di động trên $(O)$. $D$ là trung điểm $B C$. Trên $A B$ lấy các điểm $M, P$ và trên $A C$ lấy các điểm $N, Q$ sao cho $D A=D P=D Q$, dồng thời $D M \perp A C, D N \perp A B$.

(a) Chứng minh rằng các điểm $M, N, P, Q$ cùng thuộc một đường tròn $(\mathcal{C})$ và $(\mathcal{C})$ luôn đi qua một điểm cố định.

(b) Chứng minh rằng tâm của $(\mathcal{C})$ luôn thuộc một đường tròn cố định.

 

Ngày thi thứ hai

Bài 5. Cho số thực $a \neq 0$. Tìm giới hạn của dãy số $\left(u_n\right)$ thoả mãn:

$\quad\quad\quad\quad\quad\quad\quad\quad u_1=0, u_{n+1}\left(u_n+a\right)=a+1, \forall n \in \mathbb{N}^*$

Bài 6. Tìm tất cả các hàm số $f: \mathbb{R}^{+} \rightarrow \mathbb{R}^{+}$thoả mãn điều kiện:

$\quad\quad\quad\quad f\left(x f\left(y^2\right)-y f\left(x^2\right)\right)=(y-x) f(x y) \forall x, y \in \mathbb{R}^{+}, x>y$

Bài 7. Cho $n=2018.2019$. Gọi $A$ là tập hợp các bộ $\left(a_1, a_2, \ldots, a_n\right)$ có thứ tự thoả mãn điều kiện $a_i \in{0,1} \forall i \in{1,2, \ldots, n}$ và $\sum_{i=1}^n a_i=2018^2$. Có bao nhiêu bộ $\left(a_1, a_2, \ldots, a_n\right)$ từ $A$ để $\sum_{i=1}^k a_i \geq \frac{a}{2}$ và $\sum_{i=n-k+1}^n a_i \geq \frac{k}{2} \forall k \in{1,2, \ldots, n}$ ?

Bài 8. Đường tròn $(\mathcal{C})$ tâm $I$ nội tiếp tam giác $A B C$ và tiếp xúc với các cạnh $A B, A C$ tại $E, F$. $A M, A N$ là các đường phân giác trong, phân giác ngoài của góc $\angle B A C(M, N$ nằm trên $B C)$. Gọi $d_M, d_N$ lần lượt là các tiếp tuyến của $(\mathcal{C})$ qua $M, N$ và khác $B C$.

(a) Chứng minh rằng $d_M, d_N, E F$ đồng quy tại điểm $D$.

(b) Lấy trên $A B, A C$ các điểm $P, Q$ thoả mãn $D P|A C, D Q| A B$. Gọi $R, S$ là trung điểm của $D E, D F$. Chứng minh rằng $I$ thuộc đường thẳng qua các trực tâm của hai tam giác $D P S, D Q R$.

 

LỜI GIẢI

Ngày thi thứ nhất

Bài 1. Cho số nguyên $a>1$. Tìm giá trị lớn nhất của số thực $d$ sao cho tồn tại một cấp số cộng có công sai $d$, số hạng đầu tiên là $a$ và có đúng hai trong các số $a^2, a^3, a^4, a^5$ là những số hạng của cấp số cônng đó.

Lời giải: Trước hết, ta chứng minh rằng $d=a^3-a$ thoả mãn điều kiện. Thật vậy, xét cấp số cộng có số hạng đầu là $a$ và công sai là $d=a^3-a$ thì

$\quad\quad\quad\quad\quad\quad\quad\quad\quad\quad \left\{\begin{array}{l}a^3=a+\left(a^3-a\right) \\ a^5=a+\left(a^3-a\right)\left(a^2+1\right)\end{array} .\right.$

Do đó $a^3, a^5$ cùng thuộc cấp số cộng có công sai $d=a^3-a$.

Giả sử rằng tồn tại giá trị $d>a^3-a$ thoả mãn điều kiện bài toán. Khi đó:

$\quad\quad\quad\quad\quad\quad\quad\quad\quad\quad a+d>a+a^3-a=a^3$

Dẫn đến hai số hạng thuộc cấp số cộng phải là $a^4$ và $a^5$. Lại để ý rằng $a>1$ nên có $a<a^4<a^5$, kết hợp lại thì phải tồn tại hai số nguyên dương $k<l$ sao cho:

$\quad\quad\quad\quad\quad\quad\quad\quad\quad\quad \left\{\begin{array}{l}a^4=a+k d \\ a^5=a+l d\end{array}\right.$

Từ đó $a(a+k d)=a+l d$ hay $d(l-a k)=a^2-a>0$.

Chú ý rằng ta có $d>0$ nên $l-a k>0$, hơn nữa $l-a k \in \mathbb{Z}$ nên $l-a k \geq 1$. Điều này dẫn đến $a^2-a \geq d>a^3-a$, vô lý do $a>1$.

Vậy giá trị lớn nhất của $d$ là $\max d=a^3-a$.

Bài 2. Cho $n$ số thực $x_1, x_2, \ldots, x_n$. Với mỗi $i \in{1,2, \ldots, n}$, gọi $a_i$ là số các chỉ số $j$ mà $\left|x_i-x_j\right| \leq 1$ và $b_i$ là số các chỉ số $j$ mà $\left|x_i-x_j\right| \leq 2(i$ và $j$ có thể bằng nhau).

(a) Chứng minh rằng tồn tại $i$ dể $b_i \leq 3 a_i$.

(b) Gọi $A$ là số cặp $(i, j)$ có thứ tự mà $\left|x_i-x_j\right| \leq 1$ và $B$ là số cặp $(i, j)$ có thứ tự mà $\left|x_i-x_j\right| \leq 2$ ( $i$ và $j$ có thể bằng nhau). Chứng minh rằng $B \leq 3 A$.

Lời giải . (a) Không mất tính tổng quát, giả sử $x_1 \leq x_2 \leq \ldots \leq x_n$.

Xét $k=\max [a_1, a_2, \ldots, a_n]$ và $a_i=k$, khi đó tồn tại $k$ số trong dãy là:

$\quad\quad\quad\quad x_u \leq x_{u+1} \leq \ldots \leq x_i \leq \ldots \leq x_v \text { với }\left|x_u-x_i\right|,\left|x_v-x_i\right| \leq 1 .$

Ngoài ra vì tính lớn nhất của $k$ nên $\left|x_{u-1}-x_i\right|>1,\left|x_{v+1}-x_i\right|>1$.

Trong $\left[x_u, x_v\right]$, có đúng $k$ chỉ số $j$ để $\left|x_j-x_i\right| \leq 1<2$. Còn trước $x_u$, xét hai số $x_r, x_s$ sao cho $x_r \leq x_s$ và $\left|x_r-x_i\right| \leq 2,\left|x_s-x_i\right| \leq 2$ thì:

$\quad\quad\quad\quad \left|x_r-x_s\right|=x_s-x_r=\left(x_i-x_r\right)-\left(x_i-x_s\right)<2-1=1$

nên sẽ có không quá $k$ số $j$ để $\left|x_j-x_i\right| \leq 2$ vì nếu ngược lại, sẽ có nhiều hơn $k$ số liên tiếp trong dãy cách nhau không quá 1 đơn vị, mâu thuẫn với tính lớn nhất của $k$. Tương tự với các số sau $x_v$, vì thế nên $b_i \leq 3 k$ kéo theo $b_i \leq 3 a_i$.

(b) Ta sẽ chứng minh bằng quy nạp theo $n$.

Với $n=1$ rõ ràng $A=B=1$ nên khẳng định hiển nhiên đúng. Giả sử kết quả đúng với $n \geq 1$, ta sẽ chứng minh nó cũng đúng với $n+1$.

Xét dãy số thực $T=\left(x_1, x_2, \ldots, x_{n+1}\right)$ bất kỳ và giả sử $x_1 \leq x_2 \leq \ldots \leq x_{n+1}$. Ký hiệu $A_T, B_T$ là số cặp có thứ tự các chỉ số $(i, j)$ tương ứng với định nghĩa của đề bài. Giả sử $k \geq 1$ là số lượng lớn nhất các số của $T$ được chứa trong một đoạn độ dài bằng 2 nào đó.

Gọi $x_i$ là số cuối cùng của dãy mà trong đoạn $\left[x_i-1, x_i+1\right]$ có chứa đúng $k$ số (kể cả $x_i$ ). Gọi $T^{\prime}$ là dãy mới sau khi bỏ $x_i$ đi. Khi đó, số lượng các số thuộc $T^{\prime}$ có trong $\left[x_i-1, x_i+1\right]$ là $k-1$, ngoài ra $x_i$ đã bị bỏ đi thuộc về đúng $2 k-1$ cặp của $A_T$.

Do đó: $A_T=A_{T^{\prime}}+2 k-1$.

Ta viết lại như sau

$\quad\quad\quad\quad \left[x_i-2 ; x_i+2\right]=\left[x_i-2 ; x_i-1\right] \cup\left[x_i-1 ; x_i+1\right] \cup\left[x_i+1 ; x_i+2\right]$

Trừ đoạn ở giữa thì hai đoạn đầu và cuối chứa tối đa $k$ phần tử của $T$. Hơn nữa, do định nghĩa số $x_i$ nên trong đoạn $\left[x_i+1 ; x_i+2\right]$ có tối đa $k-1$ phần tử của $T$. Từ đó có tối đa:

$\quad\quad\quad\quad\quad\quad\quad\quad\quad\quad\quad\quad 2(k-1)+k=3 k-2$

phần tử của $T$ (không tính $x_i$ ) thuộc $\left[x_i-2 ; x_i+2\right]$. Dẫn đến:

$\quad\quad\quad\quad\quad\quad B_T \leq 2(3 k-2)+1+B_{T^{\prime}}=3(2 k-1)+B_{T^{\prime}}$

Áp dụng giả thiết quy nạp, ta có $B_{T^{\prime}}<3 A_{T^{\prime}}$ nên từ các điều trên thì:

$\quad\quad\quad\quad B_T \leq 3(2 k-1)+B_{T^{\prime}}<3(2 k-1)+3 A_{T^{\prime}}=3\left(A_{T^{\prime}}+2 k-1\right)=3 A_T .$

Theo nguyên lý quy nạp, bài toán cũng đúng với $n+1$.

Vậy bài toán được chứng minh hoàn toàn.

Nhận xét. Bài toán này thật ra liên quan đến phương pháp xác suất trong tổ hợp, có thể xem tại quyển “The Probabilistic Method” của GS. Noga Alon. Ta xét một lời giải khác như sau:

(a) Chọn $i$ sao cho số các chỉ số $j$ để $\left|x_i-x_j\right| \leq 1$ là lớn nhất. Khi đó, số lượng chỉ số $j$ sao cho $x_j \in\left(x_i+1, x_i+2\right]$ tối đa là $a_i$, vì nếu không thì tồn tại $j$ để $a_j>a_i$. Tương tự, số lượng chỉ số $j$ sao cho $x_j \in\left[x_i-2, x_i-1\right)$ tối đa là $a_i$.

Chú ý rằng với các chỉ số $j$ để $\left|x_i-x_j\right| \leq 2$ thì ta có điều sau:

$\quad\quad\quad\quad x_j \in\left[x_i-2, x_i-1\right) \cup\left(x_i-1, x_i+1\right) \cup\left(x_i+1, x_i+2\right]$

Số lượng các chỉ số đó chính là $b_i$, dẫn đến $b_i \leq a_i+a_i+a_i=3 a_i$. Hơn nữa, nếu đẳng thức xảy ra, ta phải có mỗi đoạn (hay nửa khoảng) ở phân hoạch trên chứa chính xác $a_i$ chỉ số $j$ của $x_j$.

(b) Bài toán hiển nhiên đúng với $n=1$. Giả sử rằng tồn tại $n>1$ để kết luận không đúng, ta chọn $n$ nhỏ nhất. Ta cũng chọn $i$ sao cho $a_i$ lớn nhất.

Gọi $A^{\prime}, B^{\prime}$ tương ứng là số cặp chỉ số $(k, l)$ mà $\left|x_k-x_l\right| \leq 1$ và $\left|x_k-x_l\right| \leq 2$, trong đó $1 \leq k, l \leq n$ và $k, l \neq i$. Vì $n$ là phản ví dụ nhỏ nhất nên $B^{\prime} \leq 3 A^{\prime}$.

Các cặp chỉ số $(k, l)$ mà $k=i$ hoặc $l=i$ và $\left|x_k-x_l\right| \leq 1$ đều phải có dạng $(k, i)$ hoặc $(i, k)$ trong đó $k \neq i$ và $(i, i)$. Có tổng cộng $2\left(a_i-1\right)+1$ cặp như thế nên $A=A^{\prime}+2\left(a_i-1\right)+1$.

Tương tự thì $B=B^{\prime}+2\left(b_i-1\right)+1$. Do đó nếu $b_i \leq 3 a_i-1$ thì:

$\quad\quad\quad\quad B=B^{\prime}+2 b_i-1 \leq 3 A^{\prime}+2\left(3 a_i-1\right)-1=3\left(A^{\prime}+2 a_i-1\right)=3 A$

Điều này trái với việc $n$ là phản ví dụ nhỏ nhất. Do đó $b_i \geq 3 a_i$. Theo ý (a) thì $b_i \leq 3 a_i$, từ đây phải có $b_i=3 a_i$. Hơn nữa, số lượng chỉ số $j$ để thỏa mãn $x_j \in\left[x_i-2, x_i-1\right)$ hoặc $x_j \in\left(x_i+1, x_i+2\right]$ dều phải bằng $a_i$.

Với mỗi $j, j^{\prime}$ sao cho $x_j, x_{j^{\prime}} \in\left[x_i-2, x_i-1\right)$, ta có $\left|x_j-x_{j^{\prime}}\right|<1$, dẫn đến $a_j \geq a_i$. Mặt khác $a_i$ là lớn nhất có thể nên $a_j=a_i$. Tương tự, với mỗi $j$ sao cho $x_j \in\left(x_i+1, x_i+2\right]$ thì $a_j=a_i$. Như vậy với mọi $j$ sao cho $1<\left|x_i-x_j\right| \leq 2$ thì $a_j=a_i$. Cũng với cách chọn chỉ số $j$ đó, lập luận tương tự như những ý trên, ta cũng phải có $b_j=3 a_j$.

Xây dựng đồ thị $\mathcal{G}$ với các đỉnh được đánh số là $1,2, \ldots, n$ sao cho cặp đỉnh $(k, l)$ kề nhau khi và chỉ khi $1<\left|x_k-x_l\right| \leq 2$. Những lập luận trên cho thấy mọi đỉnh $j$ mà tồn tại một đường đi từ $i$ đến $j$ đều phải thỏa mãn $a_j=a_i$ và $b_j=3 a_j$. Gọi $\mathcal{X}$ là tập hợp tất cả các đỉnh $j$ sao cho tồn tại một đường đi từ $i$ dến $j$ trong $\mathcal{G}$. Đặt $\mathcal{Y}={1,2, \ldots, n} \backslash \mathcal{X}(\mathcal{Y}$ có thể rỗng $)$.

Bây giờ, gọi $A_y, B_y$ tương ứng là số cặp chỉ số $(k, l)$ có tính thứ tự, có thể bằng nhau mà $\left|x_k-x_l\right| \leq 1$ và $\left|x_k-x_l\right| \leq 2$, trong đó $k, l \in \mathcal{Y}$. Chú ý rằng $A_{\mathcal{Y}}=B_{\mathcal{Y}}=0$ nếu $\mathcal{Y}=\emptyset$. Bởi $n$ là phản ví dụ nhỏ nhất, ta phải có $B_{\mathcal{Y}} \leq 3 A_{\mathcal{Y}}$. Ta gọi $a_{y, k}$ và $b_{y, k}$ tương ứng là số chỉ số $j \in \mathcal{Y}$ mà $\left|x_j-x_k\right| \leq 1$ và $\left|x_j-x_k\right| \leq 2$. Định nghĩa tương tự $a_{\mathcal{X}, k}$ và $b_{\mathcal{X}, k}$.

Với mọi $k \in \mathcal{Y}$, dễ thấy $k$ không kề bất cứ đỉnh nào trong $\mathcal{X}$, vì vậy ta có được $b_{\mathcal{X}, k}=0$ và $b_k=b_{\mathcal{Y}, k}+a_{\mathcal{X}, k}$. Từ đây dẫn đến đẳng thức sau:

$\quad\quad\quad\quad\quad\quad B=\sum_{k \in \mathcal{X}} b_k+\sum_{k \in \mathcal{Y}} b_k=3 \sum_{k \in \mathcal{X}} a_k+\sum_{k \in \mathcal{Y}}\left(b_{y, k}+a_{\mathcal{X}, k}\right)$

Ta đồng thời có $\sum_{k \in \mathcal{Y}} b_{y, k}=B_{\mathcal{Y}} \leq 3 A_{\mathcal{Y}}$. Hơn nữa, ta cũng có được:

$\quad\quad A=\sum_{k \in \mathcal{X}} a_k+\sum_{k \in \mathcal{Y}} a_k=\sum_{k \in \mathcal{X}} a_k+\sum_{k \in \mathcal{Y}}\left(a_{\mathcal{Y}, k}+a_{\mathcal{X}, k}\right)=\sum_{k \in \mathbb{X}} a_k+A_{\mathcal{Y}}+\sum_{k \in \mathcal{Y}} a_{\mathcal{X}, k}$

Do đó:

$\quad\quad\quad\quad B \leq 3 A_{\mathcal{Y}}+\sum_{k \in \mathcal{Y}} a_{\mathcal{X}, k}+3 \sum_{k \in \mathcal{X}} a_k \leq 3\left(A_{\mathcal{Y}}+\sum_{k \in \mathcal{Y}} a_{\mathcal{X}, k}+\sum_{k \in \mathcal{X}} a_k\right)=3 A$

Điều này dẫn đến giả sử phản chứng là sai.

Vì vậy, với mọi số nguyên dương $n$, ta phải có $B \leq 3 A$. Bài toán kết thúc.

Bài 3. Cho $p$ là số tự nhiên. Xét phương trình nghiệm nguyên

$\quad\quad\quad\quad\quad\quad\quad\quad\quad\quad\quad\quad x^3+x+p=y^2 .$

(a) Tìm số nguyên tố $p$ nhỏ nhất dạng $4 k+1$ sao cho phương trình có nghiệm.

(b) Chứng minh rằng nếu $p$ là số chính phương thì phương trình trên có nghiệm nguyên dương.

Lời giải. (a) Các số nguyên tố có dạng $4 k+1$ là $5,13,17, \ldots$

Trước hết, ta thấy với $p=13$ thì $x^3+x+13=y^2$ có nghiệm là $(x ; y)=(4 ; 9)$. Ta sẽ chứng minh rằng phương trình $x^3+x+5=y^2$ không có nghiệm nguyên. Xét modulo 4. Có các khả năng sau xảy ra:

  • Khi $x$ chia 4 dư $0,1,2,3$, vế trái chia 4 lần lượt dư $1,3,3,3$.
  • Khi $y$ chia 4 dư $0,1,2,3$, vế phải chia 4 lần lượt dư $0,1,0,1$.

Do đó $y$ phải lẻ và $4 \mid x$. Viết biểu thức đã cho thành:

$\quad\quad\quad\quad\quad\quad\quad\quad\quad\quad (x+3)\left(x^2-3 x+10\right)=y^2+5^2$

Do $x+3 \equiv 3(\bmod 4)$ nên $x+3$ có ước nguyên tố $q \equiv 3(\bmod 4)$. Ta biết rằng với $a, b \in \mathbb{Z}$ thì $a^2+b^2$ chia hết cho số nguyên tố $q \equiv 3(\bmod 4)$ khi và chỉ khi $q \mid a$ và $q \mid b$. Từ đó thì $q \mid 5$ hay $q=5$, mâu thuẫn.

Vậy $p=13$ là số nguyên tố nhỏ nhất cần tìm.

(b) Trước hết, ta giới thiệu kết quả sau (còn gọi là định lý 4 số):

Bổ Đề. Với các số nguyên dương $a, b, c, d$ thoả mãn $a b=c d$ thì tồn tại các số nguyên dương $x, y, z, t$ sao cho $a=x y, b=z t, c=x z, d=y t$.

Chứng minh bổ đề. Đặt $k=\operatorname{gcd}(a, c)$ và viết $a=k a_1, c=k c_1$ thì rõ ràng $\operatorname{gcd}\left(a_1, c_1\right)=1$. Thay vào đề bài, ta có

$\quad\quad\quad\quad\quad\quad\quad\quad\quad\quad k a_1 b=k c_1 d \text { hay } a_1 b=c_1 d .$

Từ đây chú ý $a_1 \mid c_1 d$, nên $a_1 \mid d$, đặt $d=a_1 \ell$. Thay vào thì có $b=\ell c_1$. Từ đó, ta chọn $x=k, y=a_1, z=c_1, t=\ell$ thì có ngay điều phải chứng minh.

Quay lại bài toán, do $p$ là số chính phương nên đặt $p=a^2, a \in \mathbb{Z}$. Ta viết lại phương trình thành dạng:

$\quad\quad\quad\quad\quad\quad\quad\quad x^3+x+a^2=y^2 \text { hay } x\left(x^2+1\right)=(y-a)(y+a) .$

Áp dụng kết quả trên vào bài toán, ta thấy tồn tại các số nguyên dương $m, n, p, q$ để $x=m n, x^2+1=p q, y+a=m p, y-a=n q$. Từ đó:

$\quad\quad\quad\quad\quad\quad\quad\quad\quad\quad (m n)^2+1=p q \text { và } m p-n q=2 a \text {. }$

Xét dãy số $\left(u_n\right)$ xác định bởi $u_0=0, u_1=1, u_{n+2}=\alpha u_{n+1}+u_n$, trong đó $\alpha$ là hằng số mà ta sẽ chọn sau. Rõ ràng với mọi $n$ thì

$\quad\quad\quad\quad\quad\quad\quad\quad u_n^2-u_{n+1} u_{n-1}=(-1)^{n-1}\left(u_1^2-u_2 u_0\right)=(-1)^{n-1} .$

Khi đó, với $n$ chẵn thì $u_n^2-u_{n+1} u_{n-1}=-1$. Chọn $m n=u_{2 k}$. Ta có:

$\quad\quad\quad\quad\quad\quad\quad u_2=\alpha, u_3=\alpha^2+1, u_4=\alpha\left(\alpha^2+2\right), u_5=\alpha^4+3 \alpha^2+1$

Chọn $p=u_3, q=u_5, m n=u_4$ thì rõ ràng $(m n)^2+1=p q$. Bây giờ ta chỉ cần có được

$\quad\quad\quad\quad\quad\quad m u_3-n u_5=2 a \text { hay } m\left(\alpha^2+1\right)-n\left(\alpha^4+3 \alpha^2+1\right)=2 a .$

Từ đây chọn $\alpha=4 a^2$ và viết $m=2 a\left(\alpha^2+2\right), n=2 a\left(\alpha^4+3 \alpha^2+1\right)$ thì đẳng thức trên sẽ thoả mãn, vì

$\quad\quad\quad\quad\quad\quad\quad\quad \left(\alpha^2+1\right)\left(\alpha^2+2\right)-\left(\alpha^4+3 \alpha^2+1\right)=1 .$

Vậy phương trình có một cặp nghiệm cụ thể là

$\quad\quad (x, y)=\left(4 a^2\left(16 a^4+2\right), 2 a\left(16 a^4+2\right)\left(16 a^4+1\right)-a\right) \text { với } a=\sqrt{p} \in \mathbb{Z}^{+} .$

Bài 4. Cho tam giác $A B C$ nhọn nội tiếp đường tròn $(O)$ với $B, C$ cố định và $A$ di động trên $(O)$. $D$ là trung điểm $B C$. Trên $A B$ lấy các điểm $M, P$ và trên $A C$ lấy các điểm $N, Q$ sao cho $D A=D P=D Q$, dồng thời $D M \perp A C, D N \perp A B$.

(a) Chứng minh rằng các điểm $M, N, P, Q$ cùng thuộc một đường tròn $(\mathcal{C})$ và (C) luôn đi qua một điểm cố định.

(b) Chứng minh rằng tâm của $(\mathcal{C})$ luôn thuộc một đường tròn cố định.

Lời giải . (a) Dễ thấy tam giác $A M Q$ cân tại $M$ nên

$\quad\quad \angle D M Q=\angle D M A=90^{\circ}-\angle A=\frac{180^{\circ}-2 \angle A}{2}=\frac{180^{\circ}-\angle P D Q}{2}=\angle D P Q$

Do đó tứ giác $M P D Q$ nội tiếp. Chứng minh tương tự, ta có tứ giác $Q N D P$ nội tiếp nên $M, N, P, Q$ cùng thuộc một đường tròn $(\mathcal{C})$, và $(\mathcal{C})$ luôn đi qua điểm $D$ cố định.

(b) Gọi $K B, K C$ là hai tiếp tuyến của $(O)$. Ta có $D, K, O$ thẳng hàng, lại có:

$\quad\quad\quad\quad\quad \angle B K O=90^{\circ}-\angle B O K=90^{\circ}-\angle B A C=\angle B M D$

Từ đó tứ giác $B D K M$ nội tiếp. Để ý rằng $K D \perp B C$ nên $K M \perp A B$, hơn nữa $D N \perp A B$ nên $K M | D N$. Tương tự thì $K N | D M$. Do đó $D M K N$ là hình bình hành hay $D K, M N$ có $J$ là trung điểm chung.

Gọi $I$ là tâm của $(\mathcal{C})$ thì $I J \perp M N$ và $J L | A D$. Chú ý rằng $D$ là tâm $(A P Q)$ và cũng là trực tâm tam giác $A M N$ nên $P Q, M N$ là hai đường đối song. Đồng thời nếu $L$ là trung điểm $A D$ thì $J L$ vuông góc với đường nối hai chân đường cao từ $M, N$ của tam giác $A M N$ nên $J L \perp P Q$. Lại có $D P=D Q$ và $I P=I Q$ nên $I D \perp P Q$, do đó $J L | D I$.

Từ đây $I D L J$ là hình bình hành và $I L, D J$ có $T$ là trung điểm chung cố định. Xét phép vị tự tâm $D$ tỉ số $\frac{1}{2}$ hợp với phép đối xứng tâm $T$ thì $A \mapsto I$. Do $A$ thuộc đường tròn $(O)$ cố định nên $I$ cũng thuộc đường tròn cố định là ảnh của $(O)$ qua hợp các phép biến hình trên. Bài toán kết thúc.

Nhận xét. Bài toán này còn một hướng tiếp cận bản chất hơn như sau. Nếu gọi $A^{\prime}$ là điểm đối xứng của $A$ qua $D$ thì $K, A^{\prime}$ là hai điểm liên hợp đẳng giác trong tam giác $A B C$, từ đó đường tròn $(\mathcal{C})$ chính là đường tròn đi qua các hình chiếu của $K, A^{\prime}$ trên các cạnh tam giác $A B C$, dồng thời $I$ là trung diểm $K A^{\prime}$.

Dưới đây là một bài toán tương tự: Cho tam giác nhọn $A B C$ nội tiếp đường tròn $(O)$ có $B C$ cố định và $A$ di dộng trên $(O)$. Gọi $H$ là trực tâm tam giác và lấy điểm $E, F$ thuộc $A B, A C$ theo thứ tự đó sao cho $H$ là trung điểm $E F$.

  1. Chứng minh rằng tâm của đường tròn $(A E F)$ luôn thuộc một đường tròn cố định. Đặt là $\omega$.
  2. Giả sử $\omega$ cắt lại $(O)$ tại các điểm $X, Y$. Chứng minh rằng $X, Y, O$ thẳng hàng.

 

Ngày thi thứ hai

Bài 5. Cho số thực $a \neq 0$. Dãy số $\left(u_n\right)$ thoả mãn:

$\quad\quad\quad\quad\quad\quad\quad\quad\quad\quad u_1=0, u_{n+1}\left(u_n+a\right)=a+1 \forall n \in \mathbb{N}^*$

Tìm giới hạn của dãy số $\left(u_n\right)$.

Lời giải: Đặt $x_{n+1}=(a+1) y_n$ và $y_{n+1}=x_n+a y_n$. Ta có:

$\quad\quad\quad\quad\quad\quad\quad y_{n+2}=x_{n+1}+a y_{n+1}=a y_{n+1}+(a+1) y_n$

Đồng thời $u_n=\frac{x_n}{y_n}$. Để ý rằng $u_1=0, u_2=\frac{a+1}{a}$. Chọn $y_1=1, y_2=a$. Từ đó:

$\quad\quad\quad\quad\quad\quad\quad\quad\quad\quad y_n=\frac{(a+1)^n-(-1)^n}{a+2} \forall n \geq 1$

Công thức trên chỉ xác định với $a \neq-2$ nên xét trường hợp $a=-2$, ta có dãy

$\quad\quad\quad\quad\quad\quad\quad\quad\quad\quad \left\{\begin{array}{l}u_1=0, \\ u_{n+1}=\frac{1}{2-u_n}, n \geq 1\end{array} .\right.$

Bằng quy nạp, ta chứng minh được $u_n \in[0 ; 1)$ nên:

$\quad\quad\quad\quad\quad\quad\quad u_{n+1}-u_n=\frac{1}{2-u_n}-u_n=\frac{\left(u_n-1\right)^2}{2-u_n}>0$

Dãy $\left(u_n\right)$ tăng và bị chặn trên bởi 1 nên có giới hạn hữu hạn là $L \in(0,1)$. Giải phương trình giới hạn, ta có được $L=\frac{1}{2-L}$. Khi đó thì $L=1$.

Tiếp theo, xét $a \neq-2$, ta có:

$\quad\quad\quad\quad u_n=\frac{x_n}{y_n}=\frac{(a+1) y_{n-1}}{y_n}=\frac{(a+1)^n+(a+1)(-1)^n}{(a+1)^n-(-1)^n} \forall n \in \mathbb{N}^*$

Đặt $-(a+1)=b \in{-1 ; 1}$, ta viết lại thành:

$\quad\quad\quad\quad\quad\quad\quad\quad\quad\quad u_n=\frac{b^n-b}{b^n-1} \forall n \geq 1$

Có các khả năng sau xảy ra:

  • Nếu $b>1$ hoặc $b<-1$, tương ứng là $a<-2$ hoặc $a>0$, thì $\lim u_n=1$.
  • Nếu $-1<b<1$, tương ứng là $-2<a<0$, thì $\lim u_n=b=-(a+1)$.

Vậy ta có kết luận sau trong các trường hợp của $a$ :

  • Nếu $a \in(-2 ; 0)$ thì $\lim u_n=-(a+1)$.
  • Nếu $a \notin(-2 ; 0)$ thì $\lim u_n=-1$.

Bài 6. Tìm tất cả các hàm số $f: \mathbb{R}^{+} \rightarrow \mathbb{R}^{+}$thoả mãn diều kiện:

$\quad\quad\quad\quad\quad\quad f\left(x f\left(y^2\right)-y f\left(x^2\right)\right)=(y-x) f(x y) \forall x, y \in \mathbb{R}^{+}, x<y .$

Lời giải . Theo giả thiết thì với mọi $y>x>0$, ta đều có

$\quad\quad\quad\quad\quad\quad\quad\quad x f\left(y^2\right)-y f\left(x^2\right)>0 \Rightarrow \frac{f\left(y^2\right)}{f\left(x^2\right)}>\frac{y}{x}>1 .$

Do đó,

$\quad\quad\quad\quad\quad\quad\quad\quad y^2>x^2 \Leftrightarrow y>x \Leftrightarrow f\left(y^2\right)>f\left(x^2\right)$

nên hàm $f$ dã cho đồng biến trên $\mathbb{R}^{+}$. Trong đề bài, thay $y=x+1$, ta có

$\quad\quad\quad\quad\quad\quad\quad f\left(x f\left((x+1)^2\right)-(x+1) f\left(x^2\right)\right)=f(x(x+1))$

hay

$\quad\quad\quad\quad\quad\quad\quad\quad x f\left((x+1)^2\right)-(x+1) f\left(x^2\right)=x(x+1) $

$\quad\quad\quad\quad\quad\quad\quad\quad \Leftrightarrow \frac{f\left((x+1)^2\right)}{x+1}=\frac{f\left(x^2\right)}{x}+1, \forall x>0$

Thực hiện thao tác này nhiều lần, ta có

$\quad\quad\quad\quad\quad\quad\quad\quad \frac{f\left((x+n)^2\right)}{x+n}=\frac{f\left(x^2\right)}{x}+n, \forall x>0, n \in \mathbb{Z}^{+}$

hay

$\quad\quad\quad\quad\quad\quad\quad\quad x f\left((x+n)^2\right)-(x+n) f\left(x^2\right)=n x(x+n) .$

Trong dề bài, thay $y=x+n$, ta có

$\quad\quad\quad\quad\quad\quad f\left(x f\left((x+n)^2\right)-(x+n) f\left(x^2\right)\right)=n f(x(x+n)) $

$\quad\quad\quad\quad\quad\quad \Leftrightarrow f(n x(x+n))=n f(x(x+n)) .$

Với mọi $n \in \mathbb{Z}^{+}, y>0$, ta luôn chọn được $x>0$ để $x(x+n)=y$ nên ta có

$\quad\quad\quad\quad\quad\quad\quad\quad\quad\quad f(n y)=n f(y), \forall n \in \mathbb{Z}^{+}, y \in \mathbb{R}^{+} .$

Đặt $f(1)=a>0$, với mọi $n \in \mathbb{Z}^{+}$, cho $y=\frac{1}{n}$, suy ra

$\quad\quad\quad\quad\quad\quad\quad\quad\quad\quad f(1)=n f\left(\frac{1}{n}\right) \Rightarrow f\left(\frac{1}{n}\right)=\frac{a}{n} .$

Do đó,

$\quad\quad\quad\quad\quad\quad\quad\quad f\left(\frac{n}{m}\right)=n f\left(\frac{1}{m}\right)=\frac{n}{m} a, \forall m, n \in \mathbb{Z}^{+}$

hay $f(x)=a x, \forall x \in \mathbb{Q}^{+}$. Với mọi số thực $x_0>0$, chọn hai dãy số hữu tỷ $\left(a_n\right),\left(b_n\right)$ sao cho $a_n<x_0<b_n$ và $\lim a_n=\lim b_n=x_0$. Rõ ràng

$\quad\quad\quad\quad\quad\quad f\left(a_n\right)<f\left(x_0\right)<f\left(b_n\right) \Rightarrow a \cdot a_n<f\left(x_0\right)<a \cdot b_n,$

nên cho $n \rightarrow+\infty$, ta có $f\left(x_0\right)=a x_0$. Do đó, với mọi số thực $x>0$ thì $f(x)=a x$. Thay vào biểu thức đã cho, ta có

$\quad\quad\quad\quad\quad \left\{\begin{array}{l}f\left(x f\left(y^2\right)-y f\left(x^2\right)\right)=a^2\left(x y^2-x^2 y\right)=a^2(y-x) x y \\ (y-x) f(x y)=a(y-x) x y\end{array}\right.$

nên $a=1$. Vậy tất cả các hàm số cần tìm là $f(x)=x, \forall x>0$.

Nhận xét. Có một điều đáng chú ý ở bài toán này là việc từ giả thiết, ta phải ngầm hiểu rằng $x f\left(y^2\right)-y f\left(x^2\right)>0$ với mọi cặp số dương $x<y$. Ta có thể thêm tường minh điều kiện đó vào đề bài cho rõ. Tuy nhiên, nếu thêm theo kiểu như sau thì sẽ có một chút vấn đề phát sinh:

Tìm tất cả các hàm số $f: \mathbb{R}^{+} \rightarrow \mathbb{R}^{+}$thoả mãn với mọi cặp số dương $x<y$, nếu $x f\left(y^2\right)-y f\left(x^2\right)>0$ thì

$\quad\quad\quad\quad\quad\quad\quad\quad\quad\quad f()=(y-x) f(x y) \forall x, y \in \mathbb{R}^{+}, x<y .$

Khi đó, ta có thể nhận thêm một hàm số thỏa mãn nữa là $f(x)=\sqrt{x}$. Lý do là vì với mọi cặp số $y>x>0$, ta đều có $x f\left(y^2\right)-y f\left(x^2\right)=0$, mà vì thế, điều kiện “nếu” ở trên là sai nên mệnh đề kéo theo là đúng.

Bài 7. Cho $n=2018.2019$. Gọi $A$ là tập hợp các bộ $\left(a_1, a_2, \ldots, a_n\right)$ có thứ tự thoả mãn điều kiện $a_i \in{0,1} \forall i \in{1,2, \ldots, n}$ và $\sum_{i=1}^n a_i=2018^2$.

Có bao nhiêu bộ $\left(a_1, a_2, \ldots, a_n\right)$ từ $A$ dể:

$\quad\quad\quad\quad\quad \sum_{i=1}^k a_i \geq \frac{a}{2} \text { và } \sum_{i=n-k+1}^n a_i \geq \frac{k}{2} \forall k \in{1,2, \ldots, n}$

Lời giải. Ta giải bài toán tổng quát khi thay 2018 bởi $m \in \mathbb{Z}^{+}$. Bài toán đã cho tương đương với bài toán sau:

Trong hệ trục tọa độ Oxy, xét lưới điểm nguyên trong hình chũ nhật có đỉnh dưới bên trái là $O(0 ; 0)$ và dỉnh trên bên phải là $A\left(m^2 ; m\right)$. Dặt $B(m ; m)$ và $C\left(m^2-m ; 0\right)$, hỏi có bao nhiêu đương đi tù̀ $O \rightarrow A$ sao cho mỗi bước, ta đi sang phải hoặc lên trên 1 đơn vị, gọi là đương đi đơn, và không vượt lên trên $O B$ cũng nhu không xuống dưới $A C$ ?

Ở đây, các số $0 ; 1$ tương ứng với các bước đi lên trên, các bước đi sang phải; còn điều kiện tổng $k$ số đầu và tổng $k$ số cuối không nhỏ hơn $\frac{k}{2}$ tương ứng với số lượng bước đi lên không vượt quá số lượng bước đi sang phải. Để thuận tiện, ta gọi đường đi cắt $d$ nếu nó có các phần nằm về cả hai phía của $d$. Trước hết, ta sẽ chứng minh bổ đề sau:

Bổ Đề. Số đường đi đơn từ $O \rightarrow A(m ; n)$, có cắt đường thẳng $y=x$, là $C_{m+n}^{m+1}$.

Thật vậy, Xét đường thẳng $(d): y=x+1$, rõ ràng các đường đi đơn cắt $y=x$ dều sẽ có điểm chung với đường thẳng $(d)$ này. Tại các điểm chung đó, ta thực hiện đối xứng trục để được một đường đi mới xuất phát từ $O \rightarrow A^{\prime}(n-1, m+1)$.

Trong hình trên, đường cũ là đứt nét, còn đường mới là liền nét. Rõ ràng phép đối xứng trục trên là song ánh, biến các đường cần tìm (cắt $y=x$ ), thành các đường từ $O \rightarrow A^{\prime}$; do đó, số lượng đường cần tìm là $C_{m+n}^{n-1}$.

Trở lại bài toán,

Số đường đi đơn từ $O \rightarrow A\left(m^2 ; m\right)$ là $C_{m^2+m}^m$ vì nó bằng số cách chọn $m$ lần đi lên trong tổng số $m^2+m$ lần di chuyển, trong đó số đường đi cắt $O B$ bằng số đường đi cắt $A C$ và bằng $C_{m^2+m}^{m-1}$ (theo bổ đề).

Do đó, ta chỉ cần tìm số đường đi cắt cả $O B, A C$ với ý tưởng đối xứng hai lần đã dùng để chứng minh bổ đề.

Đầu tiên, ta thực hiện đối xứng qua đường thẳng $y=x+1$; khi đó, các đường đi đơn sẽ xuất phát từ $O \rightarrow A^{\prime}\left(m-1 ; m^2+1\right)$. Do các đường ban đầu còn vượt qua $A C$ nên các đường mới phải cắt thêm $y=x+m^2-m+3$. Tiếp tục đối xứng qua đường thẳng này, ta đưa về đếm số đường đi đơn từ $O \rightarrow A^{\prime \prime}\left(m-2, m^2+2\right)$. Suy ra số đường đi trong trường hợp này là $C_{m^2+m}^{m-2}$. Vậy theo nguyên lý bù trừ, kết quả cần tìm sẽ là

$\quad\quad\quad\quad\quad\quad\quad\quad\quad\quad C_{m^2+m}^m-2 C_{m^2+m}^{m-1}+C_{m^2+m}^{m-2} .$

Thay $m=2018$, ta có số lượng đường đi, cũng chính là số bộ thỏa mãn đề bài.

Nhận xét. Dưới đây là một số kết quả tương tự về đường đi đơn trong đề bài

$1$. Số đường đi đơn từ $(0 ; 0) \rightarrow(m ; n)$ mà không có điểm chung với $y=x$ là

$\quad\quad\quad\quad\quad\quad\quad\quad\quad\quad\quad\quad \frac{m-n}{m+n} C_{m+n}^m .$

$2$. Số đường đi đơn từ $(0 ; 0) \rightarrow(m ; n)$ mà không vượt qua $y=x$ là

$\quad\quad\quad\quad\quad\quad\quad\quad\quad\quad\quad C_{m+n}^n-C_{m+n}^{n-1} .$

$3$. Số đường đi gồm $n$ bước mà không vượt $y=x$ là

$\quad\quad\quad\quad\quad\quad\quad\quad\quad \sum_{i=n / 2}^n \frac{n !(2 i+1-n)}{(i+1) !(n-i) !}=C_n^{[n / 2]} .$

$4$. Số đường đi đơn từ $(0 ; 0) \rightarrow(m ; n)$ mà không có điểm chung với $y=x+t$ là

$\quad\quad\quad\quad\quad\quad\quad\quad\quad\quad\quad\quad C_{m+n}^n-C_{m+n}^{m-t}$.

Bạn đọc có thể dùng phương pháp tương tự trên để giải quyết các bài toán này.

Bài 8. Đường tròn $(\mathcal{C})$ tâm $I$ nội tiếp tam giác $A B C$ và tiếp xúc với các cạnh $A B, A C$ tại $E, F$. $A M, A N$ là các đường phân giác trong, phân giác ngoài của góc $\angle B A C(M, N$ nằm trên $B C)$. Gọi $d_M, d_N$ lần lượt là các tiếp tuyến của $(\mathcal{C})$ qua $M, N$ và khác $B C$.

(a) Chứng minh rằng $d_M, d_N, E F$ dồng quy tại điểm $D$.

(b) Lấy trên $A B, A C$ các điểm $P, Q$ thoả mãn $D P|A C, D Q| A B$. Gọi $R, S$ là trung điểm của $D E, D F$. Chứng minh rằng $I$ thuộc đường thẳng qua các trực tâm của hai tam giác $D P S, D Q R$.

Lời giải. (a) Gọi $X, Y$ lần lượt là tiếp điểm của tiếp tuyến thứ hai kẻ từ $M$ dến $(I)$ và $D^{\prime}$ là tiếp điểm của $(I)$ trên $B C$. Gọi $K$ là trung điểm $E F$.

Xét trong đường tròn $(I)$ thì $E F$ là đường đối cực của $A$ và $K \in E F$ nên đối cực của $K$ sẽ đi qua $A$, mà $N A \perp I A$ nên $N A$ chính là đường đối cực của $K$.

Đường đối cực của $K$ đi qua $N$ nên đối cực của $N$, là $D^{\prime} Y$, sẽ đi qua $K$. Dễ thấy rằng $A M$ là trục đối xứng của tứ giác $D^{\prime} X E F$ nên suy ra $D^{\prime} X | E F$. Xét $D^{\prime}(E F, X Y)$, ta có có $D^{\prime} Y$ đi qua trung điểm của $E F$ và $D^{\prime} X | E F$ nên

$\quad\quad\quad\quad\quad\quad\quad\quad\quad\quad D^{\prime}(E F, X Y)=-1$

hay tứ giác $E X F Y$ điều hòa. Suy ra $M X, N Y, E F$ đồng quy. Ngoài ra ta cũng có $X, Y, A$ thẳng hàng.

(b) Dễ thấy các tam giác $P E D$ và $D Q F$ là các tam giác cân. Gọi $H_1, H_2$ lần lượt là trực tâm của tam giác $\triangle D P S, \triangle D Q R$. Ta có

$\quad\quad\quad\quad\quad\quad\quad\quad \angle P H_1 S=\angle P D F=\angle A F E=\angle P E S$

nên $E P S H_1$ là tứ giác nội tiếp. Suy ra $R H_1 \cdot R P=R S \cdot R E$. Ngoài ra,

$\quad\quad\quad\quad\quad\quad\quad\quad\quad\quad K A \cdot K I=K E \cdot K F$

nên

$\quad\quad\quad\quad\quad\quad\quad\quad\quad \frac{R P}{K A} \cdot \frac{R H_1}{K I}=\frac{R E}{K E} \cdot \frac{R S}{K F} .$

Theo định lý Thales thì $\frac{R P}{K A}=\frac{R E}{K E}$ nên $\frac{R H_1}{K I}=\frac{R S}{K F}$, mà

$\quad\quad\quad\quad\quad\quad\quad R S=R D-S D=\frac{D E-D F}{2}=\frac{E F}{2}=K F$

Suy ra $R H_1=K I$, mà $R H_1 | K I$ (do cùng vuông góc với $E F$ ) nên $I K R H_1$ là hình chữ nhật, kéo theo $I H_1 | E F$. Một cách tương tự, ta có $I H_2 | E F$ vậy nên đường thẳng $H_1 H_2$ đi qua $I$.

Nhận xét. Trong câu a, tính chất $A, X, Y$ thẳng hàng của bài toán cũng đúng khi thay $M, N$ là chân các đường phân giác bởi cặp điểm liên hợp điều hòa bất kỳ với $B, C$. Điều này có được nhờ tính chất của các đường đối cực (hoặc có thể chứng minh nhờ việc sử dụng phép chiếu trực giao các chùm điều hòa).

 

 

 

 

 

 

 

 

 

 

 

 

 

 

 

 

 

 

 

 

 

 

 

 

 

 

 

 

 

 

 

 

 

 

 

 

 

 

 

 

 

 

 

 

 

 

 

 

 

 

 

 

 

 

 

 

 

 

 

 

 

 

 

 

 

 

 

 

 

 

 

 

 

 

 

 

 

 

 

 

 

 

 

 

 

 

 

 

 

 

 

 

 

 

 

 

 

 

 

 

 

 

 

 

 

 

 

 

 

 

 

 

 

 

 

 

 

 

 

 

 

 

 

 

Leave a Reply

Your email address will not be published. Required fields are marked *